Mock AIME 1 2010 Problems/Problem 3

Revision as of 07:51, 2 August 2024 by Thepowerful456 (talk | contribs) (page was empty, so i added a see also section, statement of the problem, and statement of the answer)

Problem

Let $AB$ be a line segment of length $20 \sqrt{2}$, and let $S$ be the set of all points $P$ such that $m \angle APB \geq 45^{\circ}$. Find the last three digits of the largest integer less than the area of $S$.

Solution

$\boxed{284}$.

See Also